Sunday, July 02, 2006

MRCP Part 1 Questions

Hope you find these questions useful........... For MRCP Part 1

1)Which of the following is true regarding diabetic neuropathy?
A : It is best detected by testing for light touch with cotton wool.
B : It is always bilateral.
C : It does not cause vomiting.
D : It does not cause urinary incontinence.
E : It cannot be reversed by good glycaemic control.

ANSWER: E

2) A 76-year-old man presents with an acute confusional state. He was diagnosed to have lung cancer recently.Investigations review serum calcium 3.21 mmol/l. Which is the most appropriate initial treatment?
A : Intravenous sodium pamidronate
B : Intravenous 5% dextrose
C : Oral prednisolone (20-60 mg)
D : Intravenous frusemide
E : Intravenous 0.9% sodium chloride.

ANSWER: E

3)The presence of ketoacidosis in a patient without diabetes mellitus would suggest which one of the following causes?
A : Insulinoma
B : Addisonian crisis
C : Starvation
D : Acute on chronic renal failure
E : Salicylate overdose.

ANSWER: C
To learn more about metabolic acidosis, look at my previous post of Metabolic Acidosis in MRCP!

4)A 60-year-old man presents with breathlessness. He has had a history of previous myocardial infarction and peripheral vascular disease. He has recently been started on PERINDOPRIL for hypertension. He has been anuric for 2 hour. His serum urea is 35 mmol/l and his potassium is 7 .9 mmol/l. His ECG shows tall tented T waves. The first most appropriate step in management would be:
A : ten units of soluble insulin with 50g glucose(50%) IV
B : haemodialysis
C : bicarbonate (50 mls of a 4.2% solution) by IVI
D : calcium resonium 30g
E : 10-30 ml calcium gluconate (10%) IVI.

ANSWER: E

5) Which one of these features is typical of dermatomyositis...EXCEPT?
A : Heliotrope rash around the eye
B : Gottron's papules over knuckles of fingers
C : proximal muscle weakness
D : association with malignancy
E : Sclerotic digital skin.

ANSWER: E

6)

Photo source: www.passpaces.com/MRCP/issue2.html


The 25-year-old lady is admitted with ptosis, diplopia and dysphagia. On examination he is found to have fatigable muscle weakness and
normal deep tendon reflexes. What is the most likely diagnosis?
A : Bell’s palsy
B : Guillain-Barre syndrome
C : Motor neurone disease
D : Multiple sclerosis
E : Myasthenia gravis.

ANSWER: E

No comments: